Show inequality holds by induction












0












$begingroup$


Say I have $x_1 > y_1 > 0$ and $x_{n+1} = frac{x_n + y_n}{2}, y_{n+1} = frac{2x_ny_n}{x_n + y_n}$ for $n geq 1$. Show that $x_n > x_{n+1} > y_{n+1} > y_n > 0$.



So the obvious approach to me it seems is proceed by induction.



So with our base case, $n = 1$, $x_1 > frac{x_1 + y_1}{2} > frac{2x_1y_1}{x_1 + y_1} > y_1 > 0$.



Not sure if there's a good way to clean this up but not even sure how to show this holds true for the base case.










share|cite|improve this question









$endgroup$

















    0












    $begingroup$


    Say I have $x_1 > y_1 > 0$ and $x_{n+1} = frac{x_n + y_n}{2}, y_{n+1} = frac{2x_ny_n}{x_n + y_n}$ for $n geq 1$. Show that $x_n > x_{n+1} > y_{n+1} > y_n > 0$.



    So the obvious approach to me it seems is proceed by induction.



    So with our base case, $n = 1$, $x_1 > frac{x_1 + y_1}{2} > frac{2x_1y_1}{x_1 + y_1} > y_1 > 0$.



    Not sure if there's a good way to clean this up but not even sure how to show this holds true for the base case.










    share|cite|improve this question









    $endgroup$















      0












      0








      0





      $begingroup$


      Say I have $x_1 > y_1 > 0$ and $x_{n+1} = frac{x_n + y_n}{2}, y_{n+1} = frac{2x_ny_n}{x_n + y_n}$ for $n geq 1$. Show that $x_n > x_{n+1} > y_{n+1} > y_n > 0$.



      So the obvious approach to me it seems is proceed by induction.



      So with our base case, $n = 1$, $x_1 > frac{x_1 + y_1}{2} > frac{2x_1y_1}{x_1 + y_1} > y_1 > 0$.



      Not sure if there's a good way to clean this up but not even sure how to show this holds true for the base case.










      share|cite|improve this question









      $endgroup$




      Say I have $x_1 > y_1 > 0$ and $x_{n+1} = frac{x_n + y_n}{2}, y_{n+1} = frac{2x_ny_n}{x_n + y_n}$ for $n geq 1$. Show that $x_n > x_{n+1} > y_{n+1} > y_n > 0$.



      So the obvious approach to me it seems is proceed by induction.



      So with our base case, $n = 1$, $x_1 > frac{x_1 + y_1}{2} > frac{2x_1y_1}{x_1 + y_1} > y_1 > 0$.



      Not sure if there's a good way to clean this up but not even sure how to show this holds true for the base case.







      real-analysis sequences-and-series induction






      share|cite|improve this question













      share|cite|improve this question











      share|cite|improve this question




      share|cite|improve this question










      asked Sep 28 '18 at 1:09









      SS'SS'

      597314




      597314






















          3 Answers
          3






          active

          oldest

          votes


















          0












          $begingroup$

          Your inductive hypothesis is $x_n > y_n$ then there are $3$ things to show firstly
          begin{eqnarray*}
          x_n = frac{x_n+x_n}{2} > frac{x_n+y_n}{2} > x_{n+1}.
          end{eqnarray*}

          Secondly AM-HM which follows from $(x_n-y_n)^2>0$
          begin{eqnarray*}
          frac{x_n+y_n}{2} > frac{2x_n y_n}{x_n+y_n}.
          end{eqnarray*}

          Thirdly we have $x_n y_n > y_n^2$ now add $x_n y_n$ to both sides and divide by $x_n+y_n$ and we have
          begin{eqnarray*}
          y_{n+1}= frac{2x_n y_n}{x_n+y_n}> y_n.
          end{eqnarray*}






          share|cite|improve this answer









          $endgroup$





















            0












            $begingroup$

            We don’t need induction indeed we have that



            $$x_n > x_{n+1}iff x_n =frac{x_n + x_n}{2}>frac{x_n + y_n}{2}$$



            $$x_{n+1} >y_{n+1}iff frac{x_n + y_n}{2}> frac{2x_ny_n}{x_n + y_n}iff (x_n-y_n)^2>0$$



            $$Y_{n+1} >y_{n}iff frac{2x_ny_n}{x_n + y_n}>y_niff frac{x_n+x_n}{x_n + y_n}>1$$






            share|cite|improve this answer









            $endgroup$





















              0












              $begingroup$

              Notice that for all $n$ we have $x_n,y_n>0$. Therefore if $$x_n>y_n>0$$ we have $$(x_n-y_n)^2>0\(x_n+y_n)^2>4x_ny_n\{x_n+y_nover 2}>{2x_ny_nover x_n+y_n}\x_{n+1}>y_{n+1}$$since $x_1>y_1>0$ we have $x_n>y_n>0$ for all $n$. Also $$x_{n+1}={x_n+y_nover 2}<{x_n+x_nover 2}=x_n\y_{n+1}={2x_ny_nover x_n+y_n}$$since ${2auover a+u}$ is an increasing function of $u$ for $a>0$ and $uge a$, the minimum is attained when $u=a$. Therefore $$y_{n+1}={2x_ny_nover x_n+y_n}>{2y_ny_nover y_n+y_n}=y_n$$from which we finally conclude that$$x_{n}>x_{n+1}>y_{n+1}>y_{n}$$






              share|cite|improve this answer









              $endgroup$













                Your Answer





                StackExchange.ifUsing("editor", function () {
                return StackExchange.using("mathjaxEditing", function () {
                StackExchange.MarkdownEditor.creationCallbacks.add(function (editor, postfix) {
                StackExchange.mathjaxEditing.prepareWmdForMathJax(editor, postfix, [["$", "$"], ["\\(","\\)"]]);
                });
                });
                }, "mathjax-editing");

                StackExchange.ready(function() {
                var channelOptions = {
                tags: "".split(" "),
                id: "69"
                };
                initTagRenderer("".split(" "), "".split(" "), channelOptions);

                StackExchange.using("externalEditor", function() {
                // Have to fire editor after snippets, if snippets enabled
                if (StackExchange.settings.snippets.snippetsEnabled) {
                StackExchange.using("snippets", function() {
                createEditor();
                });
                }
                else {
                createEditor();
                }
                });

                function createEditor() {
                StackExchange.prepareEditor({
                heartbeatType: 'answer',
                autoActivateHeartbeat: false,
                convertImagesToLinks: true,
                noModals: true,
                showLowRepImageUploadWarning: true,
                reputationToPostImages: 10,
                bindNavPrevention: true,
                postfix: "",
                imageUploader: {
                brandingHtml: "Powered by u003ca class="icon-imgur-white" href="https://imgur.com/"u003eu003c/au003e",
                contentPolicyHtml: "User contributions licensed under u003ca href="https://creativecommons.org/licenses/by-sa/3.0/"u003ecc by-sa 3.0 with attribution requiredu003c/au003e u003ca href="https://stackoverflow.com/legal/content-policy"u003e(content policy)u003c/au003e",
                allowUrls: true
                },
                noCode: true, onDemand: true,
                discardSelector: ".discard-answer"
                ,immediatelyShowMarkdownHelp:true
                });


                }
                });














                draft saved

                draft discarded


















                StackExchange.ready(
                function () {
                StackExchange.openid.initPostLogin('.new-post-login', 'https%3a%2f%2fmath.stackexchange.com%2fquestions%2f2933825%2fshow-inequality-holds-by-induction%23new-answer', 'question_page');
                }
                );

                Post as a guest















                Required, but never shown

























                3 Answers
                3






                active

                oldest

                votes








                3 Answers
                3






                active

                oldest

                votes









                active

                oldest

                votes






                active

                oldest

                votes









                0












                $begingroup$

                Your inductive hypothesis is $x_n > y_n$ then there are $3$ things to show firstly
                begin{eqnarray*}
                x_n = frac{x_n+x_n}{2} > frac{x_n+y_n}{2} > x_{n+1}.
                end{eqnarray*}

                Secondly AM-HM which follows from $(x_n-y_n)^2>0$
                begin{eqnarray*}
                frac{x_n+y_n}{2} > frac{2x_n y_n}{x_n+y_n}.
                end{eqnarray*}

                Thirdly we have $x_n y_n > y_n^2$ now add $x_n y_n$ to both sides and divide by $x_n+y_n$ and we have
                begin{eqnarray*}
                y_{n+1}= frac{2x_n y_n}{x_n+y_n}> y_n.
                end{eqnarray*}






                share|cite|improve this answer









                $endgroup$


















                  0












                  $begingroup$

                  Your inductive hypothesis is $x_n > y_n$ then there are $3$ things to show firstly
                  begin{eqnarray*}
                  x_n = frac{x_n+x_n}{2} > frac{x_n+y_n}{2} > x_{n+1}.
                  end{eqnarray*}

                  Secondly AM-HM which follows from $(x_n-y_n)^2>0$
                  begin{eqnarray*}
                  frac{x_n+y_n}{2} > frac{2x_n y_n}{x_n+y_n}.
                  end{eqnarray*}

                  Thirdly we have $x_n y_n > y_n^2$ now add $x_n y_n$ to both sides and divide by $x_n+y_n$ and we have
                  begin{eqnarray*}
                  y_{n+1}= frac{2x_n y_n}{x_n+y_n}> y_n.
                  end{eqnarray*}






                  share|cite|improve this answer









                  $endgroup$
















                    0












                    0








                    0





                    $begingroup$

                    Your inductive hypothesis is $x_n > y_n$ then there are $3$ things to show firstly
                    begin{eqnarray*}
                    x_n = frac{x_n+x_n}{2} > frac{x_n+y_n}{2} > x_{n+1}.
                    end{eqnarray*}

                    Secondly AM-HM which follows from $(x_n-y_n)^2>0$
                    begin{eqnarray*}
                    frac{x_n+y_n}{2} > frac{2x_n y_n}{x_n+y_n}.
                    end{eqnarray*}

                    Thirdly we have $x_n y_n > y_n^2$ now add $x_n y_n$ to both sides and divide by $x_n+y_n$ and we have
                    begin{eqnarray*}
                    y_{n+1}= frac{2x_n y_n}{x_n+y_n}> y_n.
                    end{eqnarray*}






                    share|cite|improve this answer









                    $endgroup$



                    Your inductive hypothesis is $x_n > y_n$ then there are $3$ things to show firstly
                    begin{eqnarray*}
                    x_n = frac{x_n+x_n}{2} > frac{x_n+y_n}{2} > x_{n+1}.
                    end{eqnarray*}

                    Secondly AM-HM which follows from $(x_n-y_n)^2>0$
                    begin{eqnarray*}
                    frac{x_n+y_n}{2} > frac{2x_n y_n}{x_n+y_n}.
                    end{eqnarray*}

                    Thirdly we have $x_n y_n > y_n^2$ now add $x_n y_n$ to both sides and divide by $x_n+y_n$ and we have
                    begin{eqnarray*}
                    y_{n+1}= frac{2x_n y_n}{x_n+y_n}> y_n.
                    end{eqnarray*}







                    share|cite|improve this answer












                    share|cite|improve this answer



                    share|cite|improve this answer










                    answered Sep 28 '18 at 1:25









                    Donald SplutterwitDonald Splutterwit

                    22.6k21446




                    22.6k21446























                        0












                        $begingroup$

                        We don’t need induction indeed we have that



                        $$x_n > x_{n+1}iff x_n =frac{x_n + x_n}{2}>frac{x_n + y_n}{2}$$



                        $$x_{n+1} >y_{n+1}iff frac{x_n + y_n}{2}> frac{2x_ny_n}{x_n + y_n}iff (x_n-y_n)^2>0$$



                        $$Y_{n+1} >y_{n}iff frac{2x_ny_n}{x_n + y_n}>y_niff frac{x_n+x_n}{x_n + y_n}>1$$






                        share|cite|improve this answer









                        $endgroup$


















                          0












                          $begingroup$

                          We don’t need induction indeed we have that



                          $$x_n > x_{n+1}iff x_n =frac{x_n + x_n}{2}>frac{x_n + y_n}{2}$$



                          $$x_{n+1} >y_{n+1}iff frac{x_n + y_n}{2}> frac{2x_ny_n}{x_n + y_n}iff (x_n-y_n)^2>0$$



                          $$Y_{n+1} >y_{n}iff frac{2x_ny_n}{x_n + y_n}>y_niff frac{x_n+x_n}{x_n + y_n}>1$$






                          share|cite|improve this answer









                          $endgroup$
















                            0












                            0








                            0





                            $begingroup$

                            We don’t need induction indeed we have that



                            $$x_n > x_{n+1}iff x_n =frac{x_n + x_n}{2}>frac{x_n + y_n}{2}$$



                            $$x_{n+1} >y_{n+1}iff frac{x_n + y_n}{2}> frac{2x_ny_n}{x_n + y_n}iff (x_n-y_n)^2>0$$



                            $$Y_{n+1} >y_{n}iff frac{2x_ny_n}{x_n + y_n}>y_niff frac{x_n+x_n}{x_n + y_n}>1$$






                            share|cite|improve this answer









                            $endgroup$



                            We don’t need induction indeed we have that



                            $$x_n > x_{n+1}iff x_n =frac{x_n + x_n}{2}>frac{x_n + y_n}{2}$$



                            $$x_{n+1} >y_{n+1}iff frac{x_n + y_n}{2}> frac{2x_ny_n}{x_n + y_n}iff (x_n-y_n)^2>0$$



                            $$Y_{n+1} >y_{n}iff frac{2x_ny_n}{x_n + y_n}>y_niff frac{x_n+x_n}{x_n + y_n}>1$$







                            share|cite|improve this answer












                            share|cite|improve this answer



                            share|cite|improve this answer










                            answered Sep 28 '18 at 1:24









                            gimusigimusi

                            92.9k94494




                            92.9k94494























                                0












                                $begingroup$

                                Notice that for all $n$ we have $x_n,y_n>0$. Therefore if $$x_n>y_n>0$$ we have $$(x_n-y_n)^2>0\(x_n+y_n)^2>4x_ny_n\{x_n+y_nover 2}>{2x_ny_nover x_n+y_n}\x_{n+1}>y_{n+1}$$since $x_1>y_1>0$ we have $x_n>y_n>0$ for all $n$. Also $$x_{n+1}={x_n+y_nover 2}<{x_n+x_nover 2}=x_n\y_{n+1}={2x_ny_nover x_n+y_n}$$since ${2auover a+u}$ is an increasing function of $u$ for $a>0$ and $uge a$, the minimum is attained when $u=a$. Therefore $$y_{n+1}={2x_ny_nover x_n+y_n}>{2y_ny_nover y_n+y_n}=y_n$$from which we finally conclude that$$x_{n}>x_{n+1}>y_{n+1}>y_{n}$$






                                share|cite|improve this answer









                                $endgroup$


















                                  0












                                  $begingroup$

                                  Notice that for all $n$ we have $x_n,y_n>0$. Therefore if $$x_n>y_n>0$$ we have $$(x_n-y_n)^2>0\(x_n+y_n)^2>4x_ny_n\{x_n+y_nover 2}>{2x_ny_nover x_n+y_n}\x_{n+1}>y_{n+1}$$since $x_1>y_1>0$ we have $x_n>y_n>0$ for all $n$. Also $$x_{n+1}={x_n+y_nover 2}<{x_n+x_nover 2}=x_n\y_{n+1}={2x_ny_nover x_n+y_n}$$since ${2auover a+u}$ is an increasing function of $u$ for $a>0$ and $uge a$, the minimum is attained when $u=a$. Therefore $$y_{n+1}={2x_ny_nover x_n+y_n}>{2y_ny_nover y_n+y_n}=y_n$$from which we finally conclude that$$x_{n}>x_{n+1}>y_{n+1}>y_{n}$$






                                  share|cite|improve this answer









                                  $endgroup$
















                                    0












                                    0








                                    0





                                    $begingroup$

                                    Notice that for all $n$ we have $x_n,y_n>0$. Therefore if $$x_n>y_n>0$$ we have $$(x_n-y_n)^2>0\(x_n+y_n)^2>4x_ny_n\{x_n+y_nover 2}>{2x_ny_nover x_n+y_n}\x_{n+1}>y_{n+1}$$since $x_1>y_1>0$ we have $x_n>y_n>0$ for all $n$. Also $$x_{n+1}={x_n+y_nover 2}<{x_n+x_nover 2}=x_n\y_{n+1}={2x_ny_nover x_n+y_n}$$since ${2auover a+u}$ is an increasing function of $u$ for $a>0$ and $uge a$, the minimum is attained when $u=a$. Therefore $$y_{n+1}={2x_ny_nover x_n+y_n}>{2y_ny_nover y_n+y_n}=y_n$$from which we finally conclude that$$x_{n}>x_{n+1}>y_{n+1}>y_{n}$$






                                    share|cite|improve this answer









                                    $endgroup$



                                    Notice that for all $n$ we have $x_n,y_n>0$. Therefore if $$x_n>y_n>0$$ we have $$(x_n-y_n)^2>0\(x_n+y_n)^2>4x_ny_n\{x_n+y_nover 2}>{2x_ny_nover x_n+y_n}\x_{n+1}>y_{n+1}$$since $x_1>y_1>0$ we have $x_n>y_n>0$ for all $n$. Also $$x_{n+1}={x_n+y_nover 2}<{x_n+x_nover 2}=x_n\y_{n+1}={2x_ny_nover x_n+y_n}$$since ${2auover a+u}$ is an increasing function of $u$ for $a>0$ and $uge a$, the minimum is attained when $u=a$. Therefore $$y_{n+1}={2x_ny_nover x_n+y_n}>{2y_ny_nover y_n+y_n}=y_n$$from which we finally conclude that$$x_{n}>x_{n+1}>y_{n+1}>y_{n}$$







                                    share|cite|improve this answer












                                    share|cite|improve this answer



                                    share|cite|improve this answer










                                    answered Dec 1 '18 at 9:00









                                    Mostafa AyazMostafa Ayaz

                                    15.4k3939




                                    15.4k3939






























                                        draft saved

                                        draft discarded




















































                                        Thanks for contributing an answer to Mathematics Stack Exchange!


                                        • Please be sure to answer the question. Provide details and share your research!

                                        But avoid



                                        • Asking for help, clarification, or responding to other answers.

                                        • Making statements based on opinion; back them up with references or personal experience.


                                        Use MathJax to format equations. MathJax reference.


                                        To learn more, see our tips on writing great answers.




                                        draft saved


                                        draft discarded














                                        StackExchange.ready(
                                        function () {
                                        StackExchange.openid.initPostLogin('.new-post-login', 'https%3a%2f%2fmath.stackexchange.com%2fquestions%2f2933825%2fshow-inequality-holds-by-induction%23new-answer', 'question_page');
                                        }
                                        );

                                        Post as a guest















                                        Required, but never shown





















































                                        Required, but never shown














                                        Required, but never shown












                                        Required, but never shown







                                        Required, but never shown

































                                        Required, but never shown














                                        Required, but never shown












                                        Required, but never shown







                                        Required, but never shown







                                        Popular posts from this blog

                                        Plaza Victoria

                                        In PowerPoint, is there a keyboard shortcut for bulleted / numbered list?

                                        How to put 3 figures in Latex with 2 figures side by side and 1 below these side by side images but in...